Warum verschwindet das chemische Potential für Bose-Einstein-Kondensat?

Die Überlegung in einem Bose-Einstein-Kondensat besteht darin, zu versuchen, alle Teilchen in den angeregten Kontinuumszuständen durch Abstimmung des chemischen Potentials zu berücksichtigen. Allerdings bei einer kritischen Temperatur T C das chemische Potential erreicht seinen größtmöglichen Wert Null (unter der Annahme, dass der niedrigste Energiezustand des Systems Null ist), und daraus schließen wir, dass unterhalb dieser Temperatur stattdessen der niedrigste Energiezustand signifikant besetzt wird.

Allerdings scheint es hier eine Rangordnung zu geben. Wir schöpfen -zuerst- das chemische Potential aus und beginnen -dann- den niedrigsten Zustand zu besetzen. Was ist die Begründung dafür? Warum kann das niedrigste Energieniveau nicht signifikant besetzt werden, während das chemische Potential weit von Null entfernt ist?

Bearbeiten: Dies ist nicht dieselbe Frage wie das vorgeschlagene Duplikat. Diese andere Frage befasst sich nur damit, warum die Partikelzahl für das geringstmögliche chemische Potenzial maximiert wird (was imo offensichtlich ist, und das ist nicht das, was ich frage). Meine Frage ist, warum wir die Anzahl der Teilchen in den angeregten Zuständen maximieren müssen, bevor wir beginnen, den Grundzustand ernsthaft zu besetzen.

@zufällig Ich glaube nicht, dass dies ein Duplikat ist, siehe meine Änderungen
@JonathanLindgren: Wir haben einige Formatierungen verfügbar , sodass Sie verwenden können, *why*um zu erfahren, warum , anstatt den Gedankenstrich/Bindestrich hinzuzufügen.

Antworten (2)

Wir schöpfen -zuerst- das chemische Potential aus und beginnen -dann- den niedrigsten Zustand zu besetzen. Was ist die Begründung dafür?

Dieses „zuerst“/„dann“ ist nicht die Logik, die bei der Ableitung des Bose-Einstein-Kondensats verwendet wird, aber es ist ein intuitives Bild dessen, was vor sich geht.

Wir haben also für unser großkanonisches Potenzial so etwas wie: Ω = T ich Protokoll ( 1 e β ( μ ε ich ) ) , und wir können die Anzahl der Teilchen durch finden N = Ω μ . Das sind exakte, wahre Gleichungen der statistischen Quantenmechanik. Hier liegt Ihre Verwirrung:

Wir nähern uns an Ω = T v ( ε ) Protokoll ( 1 e β ( μ ε ) ) D ε , Wo v ist die Zustandsdichte. Dies ist eine gute Näherung im Fermi-Gas, wo die niederenergetischen Zustände aber nur von wenigen aus gefüllt sind N 10 23 Partikel. Dies ist eine gute Näherung für ein ideales Gas, wobei gilt: T ist groß und die meiste Beschäftigung findet in höheren Energiezuständen statt. Aber das ist eine schreckliche Annäherung für BECs, weil das Verhalten der Niedrigenergiezustände überhaupt nicht durch das Integral erfasst wird!

Was sind jetzt unsere Optionen? Wir wollen wahrscheinlich nicht direkt mit der Summe arbeiten, aber wir können nicht einfach ein Integral nehmen. Einige Bücher ziehen gerne den Grundzustandsterm heraus und approximieren den Rest mit einem Integral, und das werde ich hier tun.

Ω = T Protokoll ( 1 e β μ ) + v ( ε ) Protokoll ( 1 e β ( μ ε ) ) D ε

Dies ergibt ... (bezeichne z = e β μ )

N = z 1 z + v ( ε ) 1 z 1 e β ε 1 D ε

Bei niedrigen Temperaturen ist der Term ganz rechts grundsätzlich konstant und der Term ganz links variiert dramatisch ( z 1 z als μ 0 ), sodass Sie jeden gewünschten Wert von abgleichen können N .

Nun könnte man die Frage stellen: Bei einer endlichen Temperatur T mit Teilchenzahl N , wenn ich noch eins hinzufüge N , wie viel wird zwischen dem Grundzustand und wie viel zwischen den angeregten Zuständen aufgeteilt? Gut, wenn | μ | ist klein (dies entspricht T < T C ), z 1 β | μ | , v = C ε 1 / 2 , Und ε 1 / 2 z 1 e β ε 1 D ε = T 3 / 2 Γ ( 3 / 2 ) L ich 3 / 2 ( z ) . Mathematica sagt es mir L ich 3 / 2 ( 1 β | μ | ) ζ ( 3 / 2 ) 2 π β | μ | + Ö ( β μ ) , also insgesamt:

N z 1 z + v ( ε ) 1 z 1 e β ε 1 D ε = 1 β | μ | 1 + C T 3 / 2 Γ ( 3 / 2 ) ( ζ ( 3 / 2 ) 2 π β | μ | ) + Ö ( μ )

Für groß N und Klein μ , der Beitrag des ersten Semesters, 1 μ , dominiert mit Abstand! Es sagt Ihnen, dass "als N beginnt der Grundzustand fast alle Zustände zu halten".

Der Beitrag der vorangehenden Terme umfasst einen konstanten Term, dh die Vorstellung, dass Sie zuerst angeregte Zustände füllen, und eine Korrektur der Vorstellung, die besagt, dass Sie ein kleines bisschen der angeregten Zustände ungefüllt lassen, proportional zu | μ | .

Sie können mit diesem Zeug in unterschiedlichem Maße fortfahren. Pathria Statistical Mechanics Anhang F tut dies auf eine Weise, die nicht den ersten Term aus dem Integral herauszieht, sondern stattdessen direkt durch Schreiben mit den Summen arbeitet ε in Bezug auf die Energieniveaus eines Teilchens in einer 3D-Box.

[Bearbeiten/Fußnote]: Beachten Sie, dass die Integrale, die ich nehme, von sind ε = 0 Zu ε = . Ich gehe davon aus, dass der Grundzustand null Energie hat! Das ist in Ordnung, denn alles, was uns interessiert, ist der Unterschied μ ε ich . Ich kann wechseln ε durch jede Energie, die ich will, solange ich mich verändere μ entsprechend um die Differenz gleich zu halten. Die Wahl ε 0 = 0 gibt das Übliche μ 0 als T 0 Gesetz, aber wenn wir alles verschoben haben ε 0 = 10 J Ö u l e S oder was auch immer, hättest du sicherlich μ 10 J Ö u l e S als T 0 . Die Gleichungen wären viel hässlicher, aber die Physik wäre die gleiche.

Ich weiß, es ist ein alter Beitrag, aber ich denke, diese Antwort kann für jemanden nützlich sein.

Bose-Einstein-Statistik: N ( ϵ , μ , T ) = 1 e β ( ϵ μ )

Daraus folgt: N ( 0 , μ , T ) = 1 e β μ

Wenn N ( 0 , μ , T ) ist kleiner Respekt vor N A v , können wir die Näherung verwenden: λ N λ ρ ( ϵ ) N ( ϵ , μ , T ) D ϵ .

Nehmen wir nun an, dass die Näherung gültig ist λ N λ ρ ( ϵ ) N ( ϵ , μ , T ) D ϵ , stellen wir fest, dass das System keine niedrigere Temperatur als die Bose-Temperatur haben kann, T B .

Dies ist jedoch nicht sinnvoll, wir können sicherlich eine feste Menge Gas in einen Behälter füllen und unten abkühlen T B . Aus diesem Grund unter T B , können wir nicht davon ausgehen, dass die Annäherung gültig ist (sie würde zu unphysikalischen Ergebnissen führen). Wenn die Annäherung jedoch nicht gültig ist, bedeutet dies, dass dies der Fall ist N ( 0 , μ , T ) ist kein kleiner Respekt vor N A v (sonst wäre die Näherung gültig).

Seit, N ( 0 , μ , T ) = 1 e β μ , der einzige Weg, es für endliche Temperaturen groß zu machen, ist das μ ist nahe Null.